I need to know the answer but I need to know how to get it and work it out

I Need To Know The Answer But I Need To Know How To Get It And Work It Out

Answers

Answer 1

Let's start by completing the first table.

The questions gives the depth in two times:

- 2 hours after start -> 64 inches of water

- 4 hours after start -> 48 inches of water

From this, we can identifythe units:

- for time, we will use hours

- for depth, we will use inches

The first given values are:

- for time, 2

- for depth, 64

The second given valuesa are:

- for time, 4

- for depth, 48

So, the table is:

Quantity Name | Time | Depth of Pool

Unit | hour | inch

Given Value 1 | 2 | 64

Given Value 2 | 4 | 48

Now, we need to calculate the slope, which can be done using the given points and the equation:

[tex]\begin{gathered} m=\frac{y_2-y_1}{x_2-x_1} \\ m=\frac{48-64}{4-2} \\ m=\frac{-16}{2} \\ m=-8 \end{gathered}[/tex]

So, the slope is -8 inches per hour.

The interceptwe the b in the equation:

[tex]y=mx+b[/tex]

Where y is the depth and x is the time. So, using the first given values and the slope, we can find b:

[tex]\begin{gathered} 64=-8\cdot2+b \\ 64=-16+b \\ b=64+16 \\ b=80 \end{gathered}[/tex]

Thus, the intercept is 80 inches.

So, the formula we have for the pump is:

[tex]y=-8x+80[/tex]

Using it, we can answer questions 1 and 2:

1. After 6 hours means x = 6, so:

[tex]\begin{gathered} y=-8\cdot6+80 \\ y=-48+80 \\ y=32 \end{gathered}[/tex]

So, y = 32, thus, the depth of the water will be 32 inches.

2. The level 24 inches mean y = 24, so:

[tex]\begin{gathered} 24=-8x+80 \\ 8x=80-24 \\ 8x=56 \\ x=\frac{56}{8} \\ x=7 \end{gathered}[/tex]

Thus, x = 7, and the water will be at 24 inches after 7 hours.


Related Questions

the average daily rainfall is typically 5/8 of an inch. Last year, the average daily rainfall
was only 3/8 of an inch. What percent of the typical average daily rainfall fell last year in Green Valley?

Answers

5/3is not he the answers the answer is 6/4

Explain how travelling to a ski resort to go skiing could influence travel time

Answers

A ski trip requires a lot of planning.You must choose your travel dates, choose the ideal location, identify the best ski resorts for your family in the area, book lodging, purchase lift tickets, schedule lessons, consider your equipment and clothing needs, and possibly book flights or a rental car.

Describe the possible effects that traveling to a ski resort might have on travel time?

Beyond making reservations, there are more actions to be taken.Making decisions, locating the best offers, and making reservations take time.While some tasks should be completed in advance, others can wait till later.Before heading to the slopes, use this helpful planning timeline to help you save some green (and your sanity). Planning is more necessary when skiing with children as it differs from skiing alone.When there was an incredible snowstorm, I remember those last-minute college ski excursions that somehow came together, allowing me to jump in an SUV with someone else and crash on their lily pad with them and 10 to 20 other pals.I have happy recollections of those ski holidays when I was a younger frog. Perhaps I have forgotten the sleepless nights spent in the company of snorers and surviving on sugar and coffee, but those things were unavoidableBut now, on our mountainous excursions, Leap, my husband, who is learning to ski, and two tadpoles are also hopping along.The majority of our preferred ski resorts involve travel by plane, which is difficult to arrange at the last minute due to our tight schedules.In addition, it appears that my standards and working hours have changed, and now I no longer consider sleeping on a couch or the floor with complete strangers to be a vacation. As a result, I must think carefully and plan ahead for my ski trips in order to accommodate a family with a toddler, a tween, and adults.You should begin planning your ski vacation up to a year in advance.Choose the ski region or areas you want to go to.For instance, focus on a specific area; perhaps you want to look into the Denver region, Lake Tahoe, or Utah.Find the finest ski resort for your family by researching ski resorts in those areas.Check out mountain statistics and trail maps to determine if there are any runs that your family will enjoy.In comparison to other resorts, you might notice that one has more terrain for beginner to intermediate skiers. When there aren't any lift tickets or lessons available, it might be difficult to compare pricing during the off-season. To illustrate, imagine that you decide to go skiing in Colorado the next winter a year in advance.To help you plan and get an idea of costs so you can budget, look at the prices of different resorts in the area during peak and off-peak seasons on their websites more than a year in advance. You can also compare the costs of different resorts in the area at both peak and off-peak times.You may discover that Keystone and Breckenridge are more affordable while still providing excellent family-friendly amenities than places like Vail and Beaver Creek.

     To learn more about ski resort refer

    https://brainly.com/question/14279181

     #SPJ13

HELP PLSSS
(Attached to image)

Answers

The inequality is lies between the 5.26 < 5.71

The correct option is (a)

Given,

From the figure,

To see the number line and determine the which of the expression is true.  

What is Inequality?

Inequality is defined as an equation that does not contain an equal sign.

Inequality is a term that describes a statement's relative size and can be used to compare these two claims.

The number line goes from 5.0 to 6.0 with 9 labeled marks in between.

Each mark increases by point one

There are five labeled points.

A point labeled 5.04 is between 5.0 and 5.1.

A point labeled 5.26 is between 5.2 and 5.3.

A point labeled 5.4 is at the mark labeled 5.4

A point labeled 5.520 is between 5.5 and 5.6.

A point labeled 5.71 is between 5.7 and 5.8.

Then the ascending order of the number line will be :

5.04 < 5.26 < 5.4 < 5.52 < 5.71

Hence, The correct option is (a)i.e., 5.26 < 5.71

Learn more about Number line at:

https://brainly.com/question/13425491

#SPJ1

PLEASE HELP!!!

Explain how the cosine of an angle in the second quadrant differs from the cosine of its reference angle in the unit circle.

Answers

Answer:

the second quadrant is negative but the reference angle is exactly the same but positive

Step-by-step explanation:

looking at 150 on the unit circle in the second quadrant you will see the cosine is negative square root 3 divided by 2 but the reference angle is 30 degrees and when we look at the cosine of that which is the x value of what we see it is positive square root of 3 divided by 2

1. 3x-4=232. 9-4x=173.6(x-7)=364. 2(x-5)-8= 34Solve equations

Answers

1. 3x - 4 = 23

add 4 to both sides

3x - 4 + 4 = 23 + 4

3x = 27

Divide both sides by 3

3x/3 = 27/3

x = 9

2. 9 -4x = 17

subtract 9 from both sides

9 -9 -4x = 17 - 9

-4x = 8

Divide both sides by -4

-4x/-4 = 8/-4

x = -2

3. 6(x-7) = 36

Divide both sides by 6

x -7 = 6

add 7 to both sides

x - 7 + 7 = 6 + 7

x = 13

4. 2(x-5) -8 = 34

Expand

2x -10 -8 = 32

2x - 18 = 32

2x = 32 + 18

2x = 50

Divide both sides by 2

x =25

I need help with this question but no one is helping me! Can someone please help me​

Answers

Answer: 36.00%

60% can be written as 0.60 and/or 6/10, which simplifies to 3/5 for easier operations. The probability of 2 students both drink coffee with breakfast can be shown as 0.60 • 0.60 = 0.36. This means there is a 36% chance of both students drinking coffee with breakfast in the morning.

Step-by-step explanation:

Would you please check this answer for me? Use elimnation to solve the system of equations:2x+2y=162x+3y=14My answer: (10, - 2)

Answers

Equation

[tex]\begin{gathered} 2x+2y=16 \\ 2x+3y=14 \\ \end{gathered}[/tex]

Let's multiply -1 by Eq 1

[tex]\begin{gathered} -2x-2y=-16 \\ 2x+3y=14 \\ 2x-2x+3y-2y=-16+14 \\ y=-2 \end{gathered}[/tex]

Now for x

[tex]\begin{gathered} 2x+2y=16 \\ x=\frac{16-2y}{2} \\ x=\frac{16-2\cdot(-2)}{2} \\ x=\frac{16+4}{2} \\ x=\frac{20}{2} \\ x=10 \end{gathered}[/tex]

The answer would be (10, -2)

Convert 39 yards into miles. Round your answer to the nearest hundredth.

Answers

The conversion from yards to miles is given by:

[tex]1\text{ mile }\to1760\text{ yards}[/tex]

The conversion factor is:

[tex]\frac{1\text{ mile}}{1760\text{ yards}}[/tex]

Now, if we want to know how many miles are 39 yards, we multiply 39 yards by the conversion factor:

[tex]39\text{ yards }\cdot\frac{1\text{ mile}}{1760\text{ yards}}\approx0.02\text{ miles}[/tex]

HELP PLEASE !!!!!!!!!!!!!!!!!!!!!!!!!!

Answers

The equation of the line perpendicular to y = - (2 / 3) · x and the point (4, - 8) is y = (3 / 2) · x - 14.

How to determine the equation of a perpendicular line

Mathematically speaking, lines are represented by equations of the form y = m · x + b, where m and b are the slope and the intercept of the line. Additionally, two lines are perpendicular to each if the product of their slopes is equal to - 1.

First, we determine the slope of the perpendicular line:

m' = - 1 / m

m' = - 1 / (- 2 / 3)

m' = 3 / 2

Second, the intercept of the perpendicular line is:

b = y - m · x

b = - 8 - (3 / 2) · 4

b = - 8 - 6

b = - 14

Then, the equation of the perpendicular line is y = (3 / 2) · x - 14.

To learn more on perpendicular lines: https://brainly.com/question/17565270

#SPJ1

Carrots costs $0.99 per pound. Broccoli costs $1.50 per pound. Jack buys 3.5 pounds of carrots and 1.25 pounds of broccoli. What is Jack’s total bill?

Answers

Answer: his total bill is 7.24

Step-by-step explanation:

The graph shows the linear parent function.Y5-55----Which statement best describes the function?

Answers

From the picture, we have the graph of the linear parent function f(x) = x.

We have the following statements as descriptions of the function:

A. The function is negative when x < 0.

From the graph, we see that the function takes negative values for x < 0. This statement is true.

B. The function is negative when x < 0 and also when x > 0.

The first part of this statement is true, but the second is not because that we see that the function takes positives values for x > 0. So this statement is false.

C. The function is never negative.

If we see the graph, the function is negative when x < 0. So this statement is false.

D. The function is negative when x > 0.

Again, seeing the graph we note that the function takes positive values for x > 0. So this statement is false.

So the only statement that it is true, is option A.

A bag contains 120 marbles. Some are red and the rest are black. There are 19 red marbles for every black marble. How many red marbles are in the bag?

A bag contains 120 marbles. Some are red and the rest are black. There are 19 red marbles for every black marble. How many red marbles are in the bag?

Answers

The number of red marbles in the bag is 114 .

in the question ,

it is given that ,

total number of marbles in the bag = 120 marbles

let the number of red marbles [tex]=[/tex] r

let the  number of black marbles [tex]=[/tex] b

so , According to the question

b + r = 120   ...eqn(1)

also given that for every black marble there is 19 red marbles

which is represented by , r = 19b

Substituting  r = 19b in eqn (1) , we get

b + 19b = 120

simplifying further , we get  

20b = 120

b = 6

and r = 19(6) = 114

Therefore , The number of red marbles in the bag is 114 .

Learn more about Equations here

https://brainly.com/question/11461055

#SPJ1

4. What common characteristics will two parabolas in the family f(×) = a(× - 4)(× - 8) have?

Answers

• The Same roots

,

• The same axis of symmetry

1) If we compare two parabolas from this family, like:

[tex]\begin{gathered} f(x)=-1(x-4)(x-8) \\ g(x)=2(x-4)(x-8) \end{gathered}[/tex]

Note that the difference is in the leading coefficient "a".

2) Both parabolas will have the same roots, the same x-intercepts:

[tex]x_1=4,x_2=8[/tex]

In addition to this, we can state that both will share the same axis of symmetry:

[tex]\begin{gathered} f(x)=-1(x-4)(x-8) \\ f(x)=\quad -x^2+12x-32_{} \\ h=\frac{-12}{2(-1)}=6 \end{gathered}[/tex]

As well as:

[tex]\begin{gathered} g(x)=2(x-4)(x-8) \\ g(x)=2x^2-24x+64 \\ h=\frac{24}{2(2)}=6 \end{gathered}[/tex]

Hence, the answer is:

0. Same roots

,

1. Same axis of symmetry

The area of a trapezoid is A = 12 (b 1 + b 2)h. Which equation represents the area of a trapezoid when solved for b 1?

Answers

If the area of a trapezoid is A = [tex]\frac{1}{2} (b_{1}+ b_{2})h[/tex], then the equation represents the value of [tex]b_{1}[/tex] is [tex]b_{1}=\frac{2A}{h}-b_{2}[/tex]

The trapezoid is a quadrilateral with at least one pair of parallel sides is called trapezoid. There are four vertices and four edges in the trapezoid.

The area of the trapezoid  A = [tex]\frac{1}{2} (b_{1}+ b_{2})h[/tex]

Where [tex]b_{1}[/tex] and [tex]b_{2}[/tex] are the length of the parallel sides of  the trapezoid

h is the height of the trapezoid

A = [tex]\frac{1}{2} (b_{1}+ b_{2})h[/tex]

Move 1/2  to other side

[tex](b_{1}+ b_{2})h[/tex] = 2A

Move h to other side

[tex](b_{1}+ b_{2})[/tex]  = [tex]\frac{2A}{h}[/tex]

Move [tex]b_{2}[/tex]  to other side

[tex]b_{1}=\frac{2A}{h}-b_{2}[/tex]

Hence, if the area of a trapezoid is A = [tex]\frac{1}{2} (b_{1}+ b_{2})h[/tex], then the equation represents the value of [tex]b_{1}[/tex] is [tex]b_{1}=\frac{2A}{h}-b_{2}[/tex]

Learn more about trapezoid here

brainly.com/question/7328615

#SPJ1

help meeeeeeeeeeeeeeeeeeeeeeeeeeeeeeee

Answers

Answer:

-0.07

Step-by-step explanation:

Find the average

One-third of a number is at least four less than the number. Which inequality represents the number?

Answers

When one-third of a number is at least four less than the number, the inequality to represent the information is C. n ≥ 6.

What is an inequality?

It should be noted that an inequality is simply used to illustrate the expressions that aren't equal.

Let the number be n.

This will be illustrated as:

1/3n ≥ n - 4

Collect like terms

1/3n - n ≥ -4

-2/3n ≥ -4

Multiply through by -3/2

n ≥ -4 × -3/2

n ≥ 6

The value is n ≥ 6.

Learn more about inequalities on:

brainly.com/question/24372553

#SPJ1

An electronic store is having an 18% off sale on everything in the store to the nearest cent which of the following could represent regular prices and sale price of items in the store
during the sale choose all that apply

Answers

The following that represent the sales price and the regular prices are:

regular price: $55.43, sales price: $45.45

regular price: $29.99,  sales price: $24.59

regular price: $652.30, sales price: $534.89

What are the regular prices and the sales price?

Percentage is the fraction of a number out of hundred. The sign that is used to represent percentages is %.

When a discount is given on the price of an item, the price of the item would reduce by the given price or by the percent reduction.

Sales price = regular price x (1 - discount)

Sales price = regular price x (1 - 0.18)

Sales price = regular price x 0.82

Sales price when the initial price is $55.43: $55.43 x 0.82 = $45.45

Sales price when the initial price is $29.99: $29.99 x 0.82 = $24.59

Sales price when the initial price is $215.33: $215.33 x 0.82 = $176.57

Sales price when the initial price is $849.50: $849.50 x 0.82 = $696.59

Sales price when the initial price is $652.30: $652.30 x 0.82 = $534.89

Please find attached the complete question. To learn more about how to calculate discounts, please check: https://brainly.com/question/26061308

#SPJ1

David left $300 in an account that earns 5% on the money he originally deposited. When he collected his interest, he had earned $60 in interest. How long did he leave the money in the account?

Answers

The time that David left the money in the account would be = 4 years

What is an earned interest?

An earned interest is the amount of money that is regularly paid to an individual who invested a specific amount of money for a stipulated period of time.

The principal amount(p)= $300

The time (T)= X

The rate of interest yield (R)= 5 %

Simple interest( SI)= $ 60

Using the formula;

SI = P×T×R/100

Make T the subject of formula;

T = SI×100/P*R

T= 60×100/300×5

T= 6000/1500

T = 4 years.

Learn more about simple interest here:

https://brainly.com/question/25793394

#SPJ1

Suppose that the functions s and t are defined for all real numbers x as follows.
s(x)=x-3
t(x) = 3x+4
Write the expressions for (s•t) (x) and (s-t) (x) and evaluate (s+t) (-2).

Answers

7 is the value of function s and t .

What is function explain?

A relation between a set of inputs and a set of allowable outputs is called a function, and it has the property that every input is associated to exactly one output. When every element in set A has one end and only one image in set B, then the mapping from set A to set B is a function. Let A & B be any two non-empty sets.Four main categories can be used to classify the many sorts of functions. One to one function, many to one function, onto function, one to one and into function—all based on the element.

s(x)= x-3

t(x) = 3x+4

(s•t) = ( x -3 ) ( 3x + 4 )

        = ( 3x²  + 4x - 3x² - 12 )

         = 4x - 12

(s-t) (x) = ( x-3 ) - ( 3x+4 )

           =  x-3 - 3x + 4

           = -2x + 1

(s + t) (-2) =  x-3 +  3x+4

               =  4x + 1

(s + t) (-2) =  4 *  - 2 + 1

              = 8 - 1  = 7

Learn more about functions

brainly.com/question/12431044

#SPJ13

help help help help help help help

Answers

-12+6=-2/3Z
-6=-2/3z

-6*3/2=-9

Z=-9

I will need help with this math problem it has three parts to it

Answers

a)The smaller break even quantity is 10

b)The quantity that must be sold to maximize the profit= 14.25

c)The maximum profit = $36.13

STEP - BY - STEP EXPLANATION

What to find?

• The smaller break even quantity is

,

• The quantity that must be sold to maximize the profit

,

• The maximum profit.

Given:

C(x)= 13x + 370

R(x)=70x - 2x²

a) At the break even quantity, the revenue = cost, that is no profit and no loss.

R(x) = C(x)

So that we have;

70x - 2x² = 13x + 370

Re-arrange.

-2x² + 70x - 13x - 370 =0

-2x² +57x - 370 =0

2x² - 57x + 370 = 0

We can now solve the quadratic equation above.

Using the qudaratic formula

[tex]x=\frac{-b\pm\sqrt[]{b^2-4ac}}{2a}[/tex]

a=2 b=-57 c=370

Substitute the values into the formula and evaluate.

[tex]x=\frac{-(-57)\pm\sqrt[]{(-57)^2-4(2)(370)}}{2(2)}[/tex][tex]x=\frac{57\pm17}{4}[/tex][tex]Either\text{ x=}\frac{57+17}{4}=\frac{74}{4}=\frac{37}{2}[/tex]

Or

[tex]x=\frac{57-17}{4}=\frac{40}{4}=10[/tex]

x = 37/2 or x=10

We take the smaller value.

Hence, x=10

Therefore, the smaller break even quantity is 10.

b)To find the quantity that will maximize the profit, find the profit function.

p(x) = R(x) - C(x)

P(x) = 70x - 2x² - (13x +370)

= 70x - 2x² - 13x - 370

P(x) =-2x² + 57x - 370

Equate to zero

-2x² + 57x - 370 = 0

2x² - 57x + 370 =0

The maximum is at x = -b/2a

b= -57 and a=2

Substitute the values

x= - (-57) /2(2)

x= 57 /4

x= 14.25

c) To find the maximum profit, simply substitute x=14.25 into the profit function and simplify.

That is;

P(x) =-2x² + 57x - 370

P(14.25) =-2(14.25)² + 57(14.25) - 370

= -406.125 + 812.25 - 370

= 36.125

≈ 36.13

Hence, the maximum profit is $36.13

Simplify using the laws of exponents. Leave your answer an exponential form. The box is to type in the exponent.

Answers

Use the next property:

[tex]\frac{a^m}{a^n}=a^{m-n}[/tex][tex]\frac{x^8}{x^2}=x^{8-2}=x^6[/tex]Then, the given expression simplified is equal to x^6

Joshua opened a bank account with some money he had saved. During the month, he made deposits of $25 and $40, and he made withdrawals of $15 and
$35. At the end of the month, he had $170 in his account. How much money did he have when he opened the account?

A $285
B $155
C $55
D $15

Answers

Initially, Joshua had $155 in his account.

How to find the initial amount?

Deposits of $25 and $40

Withdrawals = $15, $35

At the end of the month, he had $170 in his account.

Solution:

Let x be the amount of money he had initially.

Then,

x + 25 + 40 - 15 -35 = 170

x + 65 - 50 = 170

x = 170 + 50 - 65

x = 220 - 65

x = 155

Therefore, initially, he had $155 in his account.

To learn more about the amount, refer

https://brainly.com/question/25793394

#SPJ13

(+2) The
The ratio of
women children = 11:3 and the
ratio of
men children = 5:2
Find the ratio of men:women

Answers

The ratio of both men and women 15:22

What is Ratio ?

A ratio displays the multiplicity of two numbers. For instance, if a dish of fruit contains eight oranges and six lemons, the ratio of oranges to lemons is eight to six. The ratio of oranges to the overall amount of fruit is 8:14, and the ratio of lemons to oranges is 6:8.

Given,

The ratio of men and children = 5:2

The ratio of women and children = 11:3

To find the ratio of men and women we have to divide both the ratio:

men/children/women/children

or,

men/children x children/women

putting the values

=5/2 x 3/11

=15/22

Hence, the ratio of men:women is 15:22

To learn more about Ratio click on the link

https://brainly.com/question/2914376

#SPJ9

A 20-foot support post leans against a wall, making a 70° angle with the ground. To the nearest tenth of a foot, how far is the base of the post from the wall?

Answers

The following problem can be well explained by the diagram shown below:

Hence by the trigonometry of the figure use the trigonometric ratio of cos to get:

[tex]\begin{gathered} \cos 70=\frac{BC}{AC}=\frac{x}{20} \\ x=20\cos 70 \\ x=6.8404\approx6.8 \end{gathered}[/tex]

Hence the support is 6.8 feet away from the wall.

Option A is correct.

Question 2 of 25The coach of a college basketball team categorizes the number of pointsscored by the team in each game, with the categories being 51 - 57, 58 - 64,65 - 71, and 72 or more. If the following data represent the numbers ofpoints scored in the last 10 games, how many games were in the 51 - 57category?51, 70, 63, 59, 56, 75, 60, 55, 67, 64O A1B. 2O c. 4O D. 3

Answers

Explanation

In the table, we can see that the highest number corresponds to the cats who failed. Then, if a comparative bar chart represents the data, the bar representing the number of cats that failed will be the tallest.

Answer

A. Cats who failed

whats the gcf of 96 and 40

Answers

Answer:

8

Step-by-step explanation:

Find the prime factorization of 40

40 = 2 × 2 × 2 × 5

Find the prime factorization of 96

96 = 2 × 2 × 2 × 2 × 2 × 3

To find the GCF, multiply all the prime factors common to both numbers:

Therefore, GCF = 2 × 2 × 2

GCF = 8

Answer:

8

Step-by-step explanation:

8 is the greatest number that 40 and 96 divides into

if g(x) = 1/2|x|-6 and h(x)=x-4 Which expression is equivalent to h(g(x))?

1/2|x| - 2
1/2|x-4| -6
1/2|x|-10
1/2|x-10|

Answers

h(g(x)) is equivalent to the expression 1/2|x|-10

What will the expression be?

Given g(x) = 1/2|x|-6 and h(x) = x-4

h(g(x)) = h(1/2|x|-6)              

Put the value of g(x) in the expression

Now, evaluate the expression using the  definition of h(x) by replacing x in x-4 with 1/2|x|-6:

h(g(x)) = 1/2|x|-6 - 4 = 1/2|x|-10

Since  h(g(x)) = 1/2|x|-10. Therefore, 1/2|x|-10 is the expression equivalent to h(g(x)) so the third option is the answer.

Learn more about function h(g(x))  on:

https://brainly.com/question/13048549

#SPJ1

I need help with my math

Answers

Callan hikes at the rate of 3 miles per hour, that means, the table of values of values would be completed as follows;

Hours, x Miles, y

1 3

3 9

5 15

The ordered pair above is derived by multiplying every hour by 3 to arrive at the number of miles.

The graph is now shown as;

Refer to this diagram for the next five statements. Write the correct postulate, theorem, property, or definition that justifies the statement below the diagram.

Given: ∠MRS ≅ ∠MRO

m∠MRO + m∠MRS = m∠SRO

Answers

This statement is justified by the angle addition postulate.

Other Questions
a line contains the point ( -3, -1 ) and has a slope of 1/3. which equation represents this line? Write an equation of the line that passes through (-2, 5) and (-4, -5) 1) Playing a team sport such as soccer or lacrosse can help a child learn important life skills, such as sportsmanship, loyalty, and discipline. (2) Sports also give children a chance to have fun with their friends. (3) However, being on a team can also put undue pressure on a child, canceling out the positive effects. (4) Generally though, the majority of children who play team sports have a positive experience, perform better in school, and develop strong relationships with others.Which sentence contains a rebuttal?sentence 1sentence 2sentence 3sentence 4 What kind of music should i listen to while having a Panic Attack? Find g(x), where g(x) is the reflection across the y-axis of f(x) = -9x - 4.Write your answer in the form mx + b, where m and b are integers.g(x) = Solve the following linear equation using equivalent equations to isolate the variable. Express your answer as an integer, as a simplified fraction, or as a decimal number rounded to two places. x-interecpt of 2xy+6 = 0. In a lab experiment, the decay of a radioactive isotope is being observed. At thebeginning of the first day of the experiment the mass of the substance wasand mass was decreasing by500 grams8% per day. Determine the mass of the radioactivesample at the beginning of the 13th day of the experiment. Round to the nearest tenth(if necessary). 3.Justify each step of the solution by stating the name of the property that was used to get to each step.Given: Step 1: 1.________________________________Step 2: 2. _______________________________Step 3: 17x = 403. _______________________________Step 4: 4. _______________________________Answer: El rea de un sector circular de un crculo con radio igual a 8cm y ngulo central igual a 56 es aproximadamente Unscramble the following verbs and write a sentence in Past Simple Negative. ASTDNDNUER: TICRAECP:WOLOFL: UYB: KIRDN: Which of the following is the correct mathematical expression for:The difference between three times a number and 4 i need help finding the slope, if the sides are perpendicular and if its an isosceles right triangle. #1 b) Write down the output y in terms of x.InputX+4+3Outputy6 Use fundamental identities to find the value. Step by step can u explain. Fill in the blanksDescribe and explain why a root hair cell is diffrent to a cell found in a leaf.A root hair cell has a................. Theese features maximise................. Into the root from the soil.........The leaf cell has.................. Because of theese are the parts of the cell that............. Complete the table below by writing Y for a Yes and N for a NO. The width of Davins property is 14.83 meters. An adjoining property is 19.11 meters wide. Estimate the combined width of the two properties by rounding to the nearest whole number.-------------------------------------------------------------------------------------------------------------------------------A: 31 mB: 34 mC: 34.1 mD: 33 m Put the following equation of a line into slope-intercept form, simplifying all fractions.4x-20y=-160 Name the segments in the figure below.Q R S ISelect all that apply.